0 Daumen
2,8k Aufrufe


1/12a * x^3 - x^2 + 3ax = 0


Kann mir jemand helfen bei der Funktionsschar von a > 0 ! Die Nullstellen zu finden?

x1 = 0 ist mir schon mal klar aber die anderen von a abhängig. Danke vielmal

Avatar von

5 Antworten

+2 Daumen

1/12a * x3 - x2 + 3ax = 0

Kann mir jemand helfen bei der Funktionsschar von a > 0 ! Die Nullstellen zu finden?

x1 = 0 ist mir schon mal klar aber die anderen von a abhängig. Danke vielmal

1/12 * x^2 - 2 * x + 3a = 0  | abc Formel wäre möglich oder | * 12
x^2 - 24 * x + 36a = 0  | pq-Formel oder quadratische Ergänzung
 x^2 - 24 * x + 12^2  = -36a + 144
( x - 12 )^2 = 144 - 36a
x -12 = ± ( 144 - 36a )
x =  ± ( 144 - 36a ) + 12

x = 156 - 36a
oder
x = - 132 - 36a

Voraussetzung  bei √ ( 144 - 36a )
144 - 36a >= 0

36 <= 144
a <= 4

Fallunterscheidungen

a > 4 : keine Lösung

a = 4 : 1 Lösung : x = 12 denn
x =  ± ( 144 - 36a ) + 12
x =  ± ( 144 - 36*4 ) + 12
x =  ± ( 0 ) + 12
x = 12
 
0 < a < 4 : x =  ± ( 144 - 36a ) + 12 ( siehe oben )

Avatar von 122 k 🚀

Wie kommt man denn von

1/12a * x3 - x2 + 3ax = 0


auf

1/12 * x2 - 2 * x + 3a = 0 ?

Fehlerkorrektur

1/12 * a * x2 - x + 3a = 0  | * 12/a
x2 - 12 /a *  x + 36 = 0  | pq-Formel oder quadratische Ergänzung
 x2 - 12/a * x + (6/a)^2  = -36 + 36/a^2
( x - 6/a )2 = 144/a^2 - 36
x -6/a = ± ( 36/a^2 - 36 )
x =  ± ( 36/a^2 - 36 ) + 12/a

x =   36/a^2 - 36  + 12/a
oder
x = - 36/a^2 + 36  + 12/a

Voraussetzung  bei √ ( 36/a^2 - 36 )
36/a^2 - 36 >= 0

36/a^2 >= 36
a^2 <= 1

-1 <= a <= 1

0 < a <= 1 ( Eingangsvoraussetzung )


Falls a = 1 dann 2 Nullstellen
Falls a < 1 dann 3 Nullstellen
Falls a > 1 dann 1 Nullstelle ( x = 0 )
+1 Daumen

Du machst das einfach genau so wie bei normalen Funktionen auch.  a kannst dir als Zahl vorstellen.

1/12ax^3-3ax

Wir klammern x aus und erhalten die von dir erwähnte Nullstelle.

Dann haben wir noch

1/12ax^2-3ax =0

Wir könnten jetzt umformen und die pq Formel benutzen.

Wir sehen aber direkt dass 0 wieder eine Nullstelle ist.

Dann haben wie 1/12ax-3a =0

Jetzt musst du einfach weiter umformen

Avatar von 8,7 k

Fehlerhinweis
1/12ax3-3ax 

sondern
1/12ax3 - x^2 - 3ax

Als ich geantwortet habe war das noch die Aufgabe.  Wurde wohl editiert

Das nachträgliche Editieren von Texten halte ich für schlecht
insbesondere wenn es schon Kommentare und Reaktionen
auf den Ursprungstext gibt.

Dadurch versteht man die Reaktionen nicht mehr.

mfg Georg

Das meine ich nicht.  Der Fragesteller hatte die Funktion in der Frage geändert,nachdem ich schon geantwortet hatte.

Genau das meine ich auch.

Fehlerhafter Fragetext
Deine Antwort
Korrektur des Fragetextes durch den Fragesteller
Deine Antwort stimmt nicht mehr.

0 Daumen
Hi, die Gleichung$$ \frac 1{12}\cdot a\cdot x^3 - x^2 + 3\cdot a\cdot x = 0 \text{ mit } a>0$$hat für \(0<a<1\) drei Lösungen, für \(a=1\) zwei Lösungen und für \(a>1\) eine Lösung.
Avatar von
0 Daumen

1/12·a·x^3 - x^2 + 3·a·x = 0

1/12·x·(a·x^2 - 12·x + 36·a) = 0 --> x = 0

a·x^2 - 12·x + 36·a = 0 --> x = 6/a·(1 ± √(1 - a^2)) für 0 < a ≤ 1

Avatar von 477 k 🚀
0 Daumen

ich weiß. Das Gebell aller wütenden Hunde und das Geheul der Präriewölfe wird sich über mich ergießen. Ich werde euch jetzt erklären, warum ich mit mir überein gekommen bin, dass die Mitternachtsformel ( MF ) absolut nix bringt.

Bei der Konkurrenz ===> Cosmic hatten sie so Gleichungen auch dritten Grades - und dagegen hilft keine MF mehr. Zunächst mal hast du natürlich die singuläre Lösung, die Ordinate x = 0 .  Wenn wir die mal weg dividieren, bleibt eine quadratische Gleichung



1/12  a  x  ²  -  x  +  3  a  =  0     (  1  )



Allen diesen Modellen ist nämlich eines gemeinsam: Zwar handelt es sich um Polynome in x ; der Parameter a geht aber nur linear ein. Und da kam ich schließlich auf den Trichter, alles nach a umzustellen und nicht nach x ; das können wir nämlich:



a  =  12  x  /  (  x  ²  +  36  )      (  2  )


Im Grunde ist  ( 2 ) ein ===> Nomogramm; um die Nullstellen zu bekommen, legst du das Lineal im Abstand a an die Ordinate und schaust, wo sich Schnittpunkte mit der Kurve ergeben.

Dass in ( 2 ) irgendwas besser sein muss als in ( 1 ) , sieht man auf einen Blick. ( 2 ) ist eine gebrochen rationale Funktion und als solche den Mitteln der Kurvendiskussion zugänglich. Z.B. wäre ja rein teoretisch denkbar, dass ( 2 ) Polstellen besitzt; eine Fragestellung, an die du mit der MF nie ran kommst: Gibt es x, welche niemals Wurzeln von ( 1 ) sein können?

Oder die Symmetrie. In ( 2 )  siehst du sofort, dass wenn du x spiegelst, kriegst du ( - a ) - ungerade Symmetrie.  In ( 1 ) würdest du direkt erst mal nicht merken, dass die Gleichung invariant bleibt, wenn du x ersetzt durch ( - x ) und gleichzeitig a durch ( - a ) D.h. wegen deiner Forderung a > 0  bewegen wir uns wohl oder übel auf dem rechten Ast des Nomogramms.

Was würde in ( 1 )  passieren, wenn du die Mitternachtswurzel ziehst? Du vertauschst die Achsen, wie wenn du das Gemälde einer Landschaft so drehst, dass der Erdboden vertikal verläuft - selber raus plotten und ausprobieren ... Ist es nicht bemerkenswert, dass die Ästetik einer matematischen Formel genau das wider spiegelt, was wir intuitiv als Schönheit ansprechen? Wurzeln sind doch nur das, was die Angelsachsen als " alkward " bezeichnen.

Alles was wir doch wissen wollen: Für welche a gibt es Null, eine bzw. zwei Lösungen? Erledigen wir, indem wir die Extrema von ( 2 ) aufsuchen.

Zunächst die Grobskizze mit Asymptotik; da der Nennergrad ( 2 )  größer ist als der Zählergrad ( 1 ) verebbt der Graf bei x = ( °° ) in ( + 0 )  x = 0 ist Nullstelle - wir erwarten ein Maximum .

Gebrochene Terme Ableiten ist nicht so mein Ding; bringen wir ( 2 ) erst mal auf ganz Rational:



a  (  x  ²  +  36  )  =  12  x     (  3a  )


Zum Einsatz kommt ===> implizites Differenzieren; wir leiten ( 3a ) nach der Produktregel ab, wobei wir die Ableitung a ' sang-und klanglos Null setzen:


2  a  x  =  12  ===>  a  x  =  6   (  3b  )


Du kennst das Additions-und das Subtraktionsverfahren; kennst du auch das Divisionsverfahren?  ( 3a ) : ( 3b ) , um uns des Parameters a zu entledigen:



(  x  ²  +  36  )  /  x  =  2  x   ===>   x  (  max  )   =  6     (  4a  )

a  (  max  )  =  1     (  4b  )


Zunächst einmal a = 0 ; da hatten wir doch gesagt,  ( 1 ) hat nur die triviale Lösung. Die zweite war ja asymptotisch uneigentlich x2 = ( °° )  In dem Intervall   0 < a < 1  bekommst du zwei Schnittpunkte mit dem Nomogramm; schließlich bei a = 1 hast du eine Doppelwurzel x1;2 = 6 mit ( horizontaler ) Tangente . Bei a > 1 sind keine reellen Lösungen mehr möglich; dieser Graf ist in einem y-Streifen beschränkt.

Avatar von 1,2 k

Ein anderes Problem?

Stell deine Frage

Willkommen bei der Mathelounge! Stell deine Frage einfach und kostenlos

x
Made by a lovely community